1answer.
Ask question
Login Signup
Ask question
All categories
  • English
  • Mathematics
  • Social Studies
  • Business
  • History
  • Health
  • Geography
  • Biology
  • Physics
  • Chemistry
  • Computers and Technology
  • Arts
  • World Languages
  • Spanish
  • French
  • German
  • Advanced Placement (AP)
  • SAT
  • Medicine
  • Law
  • Engineering
mylen [45]
2 years ago
7

The product of a binomial and a trinomial is x3+3x2-x+2x2+6x-2 Which expression is equivalent to this

Mathematics
1 answer:
solong [7]2 years ago
8 0
The answer is the first one.
Explanation:
X^3 stays the same because there are no other cubed numbers in the problem
Next you combine the x^2s
The x^2s are +3x^2 and +2x^2
Since they are both positive, you add them: 3x^2 + 2x^2 = 5x^2
Next you do the x values
-x and +6x, also known as 6x - x = 5x
Lastly, you just add in the -2 and get:
X^3 + 5x^2 + 5x - 2
You might be interested in
Hello, Brainly community!
ioda

Answer:

(B)  \displaystyle \frac{W(3.1) - W(2.9)}{0.2}

General Formulas and Concepts:

<u>Calculus</u>

Limits

Derivatives

  • The definition of a derivative is the slope of the tangent line.

Derivative Notation

Instantaneous Rates

  • Tangent Line: \displaystyle f'(x) = \frac{f(b) - f(a)}{b - a}

Step-by-step explanation:

Since we are trying to find a <em>rate</em> at which W(t) changes, we must find the <em>derivative</em> at <em>t</em> = 3.

We are given 2 close answer choices that would have the same <em>numerical</em> answer but different <em>meanings</em>:

  1. (A)  \displaystyle  \lim_{t \to 3} W(t)
  2. (B)  \displaystyle \frac{W(3.1) - W(2.9)}{0.2}

If we look at answer choice (A), we see that our units would simply just be volume. It would not have the units of a rate of change. Yes, it may be the closest numerically correct answer, but it does not tell us the <em>rate</em> at which the volume would be changing and it is not a derivative.

If we look at answer choice (B), we see that our units would be cm³/s, and that is most certainly a rate of change. Answer choice (B) is also a <em>derivative</em> at <em>t</em> = 3, and a derivative tells us what <em>rate</em> something is changing.

∴ Answer choice (B) will give us the best estimate for the value of the instantaneous rate of change of W(t) when <em>t</em> = 3.

Topic: AP Calculus AB/BC (Calculus I/I + II)

Unit: Differentiation

Book: College Calculus 10e

8 0
2 years ago
How do you solve 2/3x+15=17? it is confusing. i have been stuck on this problem for a while...
kirill [66]
Find the opposite of adding 15. So it would be subtracting 15. So subtract 15 on both sides. So it would have 2/3x on the left and then 2 on the other side. (2/3x=2) Now find the opposite of multiplying 2/3. It is dividing 23/3. So divide 23/3 on both sides. So multiply 2/3 times 1/2. It would equal 3. x = 3
Hope this helps. :-)
3 0
3 years ago
Can someone give me the answer
larisa [96]

change it to 269 then add 15

5 0
2 years ago
Read 2 more answers
Please help 18939290201010
aalyn [17]

Answer: 3 3/5

3 3/4=3.75

3 3/5=3.6

3.6 is less than 3.75

6 0
3 years ago
If one face of a cube has a surface area of 5.25 square meters, what is the surface area of the entire cube
natali 33 [55]

Answer:

11.25

Step-by-step explanation:

if a cube has 6 equal sides, you want to multiply 5.25 by 6 to get the surface area for the entire cube.

6 0
3 years ago
Other questions:
  • Paula has a digas that weighs 3 times as much as Carla's. The total weight of the dogs is 48 lb how much does Paula dog weigh. D
    15·2 answers
  • Help me with number 6 and 4 please
    10·1 answer
  • HELP PLEASE<br><br> what is the slope of the function
    9·1 answer
  • 2x - 3y=-6<br> -5x - 9y = 15<br><br> Elimination with multiplication
    6·1 answer
  • ^ question :)))))))))))
    5·1 answer
  • If 3 suspects who committed a burglary and 6 innocent persons are lined up, what is the probability that a witness who is not su
    9·1 answer
  • What is the next fraction in this sequence? Simplify your answer.<br> 5/6, 2/3, 1/2, 1/3, ...
    11·2 answers
  • PLSSSSSSS HELPPPPPPP I WILL GIVE BRAINLIESTTTTTTTTTT!!!!!!!!!!!!!!!!!!!!!PLSSSSSSS HELPPPPPPP I WILL GIVE BRAINLIESTTTTTTTTTT!!!
    12·1 answer
  • Plsssssssss help!!!!!!!!!!!!!!!!!
    15·2 answers
  • The segment GH and GI are tangent
    8·1 answer
Add answer
Login
Not registered? Fast signup
Signup
Login Signup
Ask question!